LSAT and Law School Admissions Forum

Get expert LSAT preparation and law school admissions advice from PowerScore Test Preparation.

 James Finch
PowerScore Staff
  • PowerScore Staff
  • Posts: 943
  • Joined: Sep 06, 2017
|
#45896
Hi erust2,

Absolutely! The first step to zeroing in on the correct answer choice is to immediately eliminate the Loser answer choices and then decide between remaining Contenders. Often this, will leave only two, and a second, more careful read through of them will show one to be a clear winner over the other. Other times, eliminating all the other answer choices only leaves one possibility; you may not love it, especially at first, but if it is the only one that could conceivably answer the question, then it is the correct answer choice.

Hope this helps!
 ksikanon
  • Posts: 13
  • Joined: Apr 14, 2018
|
#47375
Is "almost all" the same as "most?" or is it more like "most, but possibly all?"

I'm struggling with B because let's say in a given scenario, we have 1,000 scientists who take the survey. 600 accept W-L, and 501 accept B-E. For the conclusion to be true, you need to have over 500 scientists who are from the W-L group and 500 from the B-E group. So, while I see why (B) is incorrect due to the word "exactly," had that answer choice said something like "Some of the scientists in the survey who know the B-E experiment are the same ones who accept W-L." Would that be a necessary assumption?
 James Finch
PowerScore Staff
  • PowerScore Staff
  • Posts: 943
  • Joined: Sep 06, 2017
|
#47601
Hi Ksikanon,

"Almost all" is generally used on the LSAT as a stand-in for "all." Here, the formal logic isn't the issue--two subgroups that each comprise "almost all" of a larger group, when combined, would definitely make up "most" of that larger group. The problem is whether it is logical to combine the two groups based on what we know; here, we have an issue of whether the scientists understand or are even aware of the Minsk Hypothesis (MH). Without knowing how much the scientists know about the MH and how it is contradicted by the combination of Wang's Law and the Brown-Eisler Experiment, we can't infer their attitude towards it.

Hope this helps!
 gcs4v333
  • Posts: 19
  • Joined: Oct 09, 2018
|
#59808
I've read all of the explanations and I still don't understand why (A) is correct. I don't think any other answer choice is more correct-seeming, but usually after I read the explanations provided here I have an "A-ha" moment, but I'm not getting it here.

In my head I diagrammed the question as:

Premise: A large survey of scientists found that almost all (scientists surveyed):
A) Accept Wan'gs Law and
B) Know the results of the Brown-Eisler Experiment

Premise: The results of the Brown-Eisler Experiment + Wang's Law contradict the Minsk Hypothesis

Conclusion: Most of the scientists surveyed reject he Minsk Hypothesis.

I don't understand why being aware that the results of B-E + WL is necessary to reach the conclusion. Can't I reject a hypothesis before I know of the hypothesis' existence? To make perhaps a bad analogy: If I accept the Law of Gravity and I know the results of Newton dropping a couple of balls off the roof, and there's an Gravity Isn't True Hypothesis floating around, don't I reject it without even knowing about it?

Maybe "reject" is too strong of a verb. If I "reject" something I have to be aware of the thing-I'm-rejecting's existence. But Answer Choice (A) isn't saying "The scientists are aware of MH's existence" it's saying they have to actively know B-E + WL contradict MH. And I still don't see why they have to know that.

After typing all of this out, I'm even more confused than I was before. Can someone please diagram this problem and map out exactly how (A) completes a piece of the puzzle that is missing? Thank you.
 Malila Robinson
PowerScore Staff
  • PowerScore Staff
  • Posts: 296
  • Joined: Feb 01, 2018
|
#59828
Hi gcs4v333,
It sounds like your reasoning is right where you need it to be. As you stated, the conclusion is that most of the scientists who were surveyed reject the Minsk Hypothesis. (And as you reasoned, a rejection requires that you know about something and purposely dismiss it.)
But the only proof we are given for that rejection is that there is a contradiction between the combination of the Brown-Eisler Experiment results & Wang’s Law (which most of those same scientists accept) and the Minsk Hypothesis.
If the scientists were not aware of this contradiction then we couldn't automatically jump to the conclusion that they reject the Minsk Hypothesis. We could say "Therefore it is likely that most of the scientists who were surveyed would reject the Minsk Hypothesis." But this conclusion is stating that they definitely reject it, and that requires them to be aware of the contradiction. That leads to Answer A as the correct answer.
Hope that helps!
-Malila
 Res Publica
  • Posts: 11
  • Joined: Aug 30, 2018
|
#66405
I'm getting stuck on B. I looked to formal logic to answer this and diagrammed it as such:




S :most: W (most scientists accept Wang's law
S :most: BE ( most scientists know Brown-Eisler results)

W + BE :dblline: M ( Wang and Brown-Eisler are incompatible with Minsk Hypothesis).

//
S :most: ~M

using FL, I get to the setup BE <---M--- S ---M---> W
I get BE :some: W. In my eyes, answer choice B solved this problem from jumping to "some" to "most" by stating that BE :dbl: W, which allows us to get S :most: M.

Where am I going wrong? Obviously I didn't diagram this but this was my thought process: there is a jump from two most statements to a most statement. To get to a most statement from two most statements, we need an assumption that they are overlapping entirely. Granted, FL was obviously the wrong way to go here. But how do you know when to use FL and when not to? Thanks.

Additionally, I took "almost all" as "most." Should I treat this as "all" or "most" from now on?
 Jeremy Press
PowerScore Staff
  • PowerScore Staff
  • Posts: 1000
  • Joined: Jun 12, 2017
|
#66693
Hi Res,

Great question!

Generally, I agree with the way you've diagrammed the formal logic relationships in the question, though as you note I don't think it's necessary to diagram in order to get where you need to be. My biggest quibble with your diagrams: the symbol W from the first sentence is not "Wang's Law" but rather "Accept Wang's Law." BE in the first sentence is not "Results of Brown-Eisler" but rather "Know Results of Brown-Eisler." That's a subtle difference, but important. It means you can't automatically make the diagrammatic connection you made between Scientists and the Minsk Hypothesis. Indeed, it's the scientists' knowledge/awareness that's missing from the second sentence, and that is required to bridge the gap to those scientists' (intellectual, i.e. "knowing") rejection of the Minsk hypothesis in the conclusion.

What turns out to be wrong with answer choice B is the word "exactly," and your mistake is in treating "almost all" the same as "most." What LSAC means conceptually by "almost all" is exactly that: close to all. It's a term that, in LSAC's usage, has a quantitative connotation quite a bit greater than "most." How much greater than most? That's a little fuzzy. In my view, certainly greater than 75%, and probably more like greater than 95%. Thus, you should probably avoid representing the notion of "almost all" diagrammatically as "most" (and definitely don't represent it as "all," because that would lead to serious problems). Maybe use the connector "AA" in the arrow to represent the "almost all?"

So, what this author really needs to assume is that enough of the "almost all" scientists who accept Wang's Law are the same as the ones who know the results of Brown-Eisler, so that the author can support the "most" in the conclusion. But what if one or two of the "almost all" scientists who accept Wang's Law didn't know the results of Brown-Eisler? That wouldn't be a problem because it could still be the case that most scientists both accept WL and know BE, and we could still make the conclusion (that is, assuming they know WL + BE contradict the Minsk hypothesis). Thus, answer choice B is not necessary.

I hope this helps!

Jeremy
 stu(dying)
  • Posts: 7
  • Joined: Aug 18, 2019
|
#72472
Whenever I see "survey" in a question, I automatically question it - is the sample representative? etc.

In this stimulus, I noticed it was a NA, but I still thought maybe the scientists that know the results and the scientists that accept Wang's Law are two different groups - therefore, it would not mean that most scientists reject the Minsk hypothesis.

This is where I feel like I was trapped into answering B on this question.

Looking at the forum, I understand how B is too strong and how A is correct. However, I want to make sure that I don't make this mistake again.

Should I still question the stimulus if it mentions survey results (on questions that are not Flaw in Reasoning)?
 Adam Tyson
PowerScore Staff
  • PowerScore Staff
  • Posts: 5191
  • Joined: Apr 14, 2011
|
#72483
Great username, stu(dying)!

You should always be wary of surveys on the LSAT, and keep in mind that the sample may not be representative of a larger group, and that the survey could have been poorly constructed (biased or ambiguous questions), and that the survey respondents may not have given accurate, truthful answers. That could lead you to formulate a good prephrase for a Strengthen, Weaken, or Flaw answer, for example. In an Assumption question, if the survey data is used to support a broader conclusion, the author must assume that those problems do not exist - the group IS representative, the survey is well constructed, and the answers are accurate.

In this case, it's important to notice that the conclusion is not about ALL scientists, but only about those that were surveyed. That means there is no problem with the survey being representative, because the conclusion is only about those people surveyed, whether it is a dozen or a million scientists.

Remember to use the Negation Technique on Assumption answers when you are stuck between contenders! Negating B we get that the two "almost all" groups aren't identical, but even if that's true and a few people are in one group and not the other, the argument still holds. Negating answer A, though, wrecks the argument - nobody who is unaware of the contradiction could be expected to definitively reject the hypothesis. With that negation, there is no reason to believe ANY of them reject it! Maybe they will reject it once they find out about the problem, but for the moment they (and we) remain in the dark. Negation for the win!
 glasann
  • Posts: 61
  • Joined: Jan 07, 2020
|
#91658
I correctly chose A, but for a moment was also looking at B and wondering if this question was formal logic. At first, I thought there was going to be some kind of connector assumption that would allow you to add the two "mosts", which I know you aren't normally able to do. But what caused me to pause was that they're talking about "almost all" and not "most." Those two are different, right? "Almost all" isn't formal logic that can be used interchangeable with "most", right? I know don't if I'd call 51 out of 100 "almost all" but that's "most."

Thanks!

Get the most out of your LSAT Prep Plus subscription.

Analyze and track your performance with our Testing and Analytics Package.